Calculating Flux Using Stoke's Theorem for a Spherical Surface

  • Thread starter Thread starter sunnyday11
  • Start date Start date
  • Tags Tags
    Theorem
Click For Summary
The discussion focuses on calculating the flux of the vector field F(x,y,z) = zy i + z k through a spherical surface defined by x² + y² + z² = 4, with z ≥ √3. Two methods are explored: the line integral method, which yields a result of -√3π, and the flux of curl method, where the curl is calculated as y j - z k. The user encounters difficulties transitioning from the surface integral to a double integral using spherical coordinates and parameterization. Suggestions are made to switch to spherical coordinates for evaluating the integral and to clarify the area element dS. The conversation emphasizes the importance of proper parameterization and coordinate systems in solving the problem.
sunnyday11
Messages
14
Reaction score
0

Homework Statement



F(x,y,z) = zy i + z k

and the surface S defined by x2 + y2 + z2 = 4, z\geq\sqrt{3}



Homework Equations





The Attempt at a Solution




Using line integral method, I got -\sqrt{3}\pi

But using the flux of curl method, I got curl = y j - z k
Then I change surface integral to double integral using z=\sqrt{4 - x<sup>2</sup> - y<sup>2</sup>}
and parameterization r(t) = rcost i + rsint j.

\int ^{0}_{1}\int ^{0}_{2pi} ( r3sin2t / \sqrt{4-r<sup>2</sup>} - \sqrt{4-r<sup>2</sup>} ) dt dr


Then I arrive at \int ^{0}_{1} ( r3/\sqrt{4-r<sup>2</sup>} - \sqrt{4-r<sup>2</sup>} ) dr

and I got stuck.

Thank you very much!
 
Physics news on Phys.org
sunnyday11 said:

Homework Statement



F(x,y,z) = zy i + z k

and the surface S defined by x2 + y2 + z2 = 4, z\geq\sqrt{3}



Homework Equations





The Attempt at a Solution




Using line integral method, I got -\sqrt{3}\pi

But using the flux of curl method, I got curl = y j - z k
Good up to here, but I can't follow what you did below.
Then I change surface integral to double integral using z=\sqrt{4 - x<sup>2</sup> - y<sup>2</sup>}
and parameterization r(t) = rcost i + rsint j.

\int ^{0}_{1}\int ^{0}_{2pi} ( r3sin2t / \sqrt{4-r<sup>2</sup>} - \sqrt{4-r<sup>2</sup>} ) dt dr


Then I arrive at \int ^{0}_{1} ( r3/\sqrt{4-r<sup>2</sup>} - \sqrt{4-r<sup>2</sup>} ) dr

and I got stuck.

Thank you very much!
So you want to evaluate

\int (\nabla\times \mathbf{F})\cdot d\mathbf{S} = \int (y \hat{\mathbf{j}} - z \hat{\mathbf{k}})\cdot d\mathbf{S}

Since the surface is a piece of a sphere, I suggest you switch to spherical coordinates. What are the magnitude and direction of the area element dS?
 
Question: A clock's minute hand has length 4 and its hour hand has length 3. What is the distance between the tips at the moment when it is increasing most rapidly?(Putnam Exam Question) Answer: Making assumption that both the hands moves at constant angular velocities, the answer is ## \sqrt{7} .## But don't you think this assumption is somewhat doubtful and wrong?

Similar threads

Replies
3
Views
2K
  • · Replies 1 ·
Replies
1
Views
1K
  • · Replies 3 ·
Replies
3
Views
2K
  • · Replies 9 ·
Replies
9
Views
2K
  • · Replies 8 ·
Replies
8
Views
2K
  • · Replies 7 ·
Replies
7
Views
2K
Replies
8
Views
3K
  • · Replies 2 ·
Replies
2
Views
2K
  • · Replies 3 ·
Replies
3
Views
2K
  • · Replies 4 ·
Replies
4
Views
2K